LSAT and Law School Admissions Forum

Get expert LSAT preparation and law school admissions advice from PowerScore Test Preparation.

 Administrator
PowerScore Staff
  • PowerScore Staff
  • Posts: 8916
  • Joined: Feb 02, 2011
|
#79873
Complete Question Explanation

The correct answer choice is (C).

Answer choice (A):

Answer choice (B):

Answer choice (C): This is the correct answer choice.

Answer choice (D):

Answer choice (E):


This explanation is still in progress. Please post any questions below!
 LaCrosse
  • Posts: 13
  • Joined: Apr 09, 2016
|
#25453
Hello PowerScore Experts,

Could you help me clear up my misconceptions about this question?

This question seems to fall under the Strengthen category. What surprised me about it was the need for the test-taker to engage in an alteration of the previously established facts (aka Retroactive Continuity). Other LSAT Strengthen questions ask the test-taker to pick an answer choice that strengthens the argument by adding some new information to the already known premises. This one actually breaks the “don’t question the premises” rule without making it clear what the new rules are.

The modern critics’ current interpretation of Webster’s tragedies is: characters should be either good or evil but in Webster’s works characters possess both good and bad traits and behave “inconsistently”, thus indicating that Webster is artistically incompetent.

Which part of the above are we allowed to “adjust” in order to answer Q11? Is it Webster’s works themselves, the views of the critics, or both?

Is the right answer choice supposed to 1) show a way for the modern critics to understand why their current interpretations of Webster’s tragedies fall short, causing them to re-interpret Webster’s works more accurately; or 2) show what would have had to happen in Webster’s life in order for the modern critics’ current (as-is) view of him and his works to be correct; or 3) list such an event that would have compelled Webster to change his writing style and also force the modern critics to abandon their current view for a different one?

In more abstract terms, in the “this view would be more valid if…” type of LSAT questions, do we try to 1) find a way for the observer to change their erroneous opinion on what had happened, or 2) do we try to imagine a hypothetical in which the observer’s current (erroneous) opinion could remain the same but would no longer be erroneous, or 3) do we try to think of something that could change both the observers’ opinions as well as the actual facts that took place?

Another critical point: when an LSAT question asks us to “change the past”, should we not be concerned about the “Butterfly effect”? This passage is filled with causations in which the behaviors of individuals or groups impact how others think or act. If we change one aspect of the story, can we honestly insist that other aspects of the passage won’t inevitably change as a result?

I found this question to be a wonderful mental workout but I am uncertain just how to most effectively attack a problem as quirky as this one on the test.


Thank you,

Alex
 Adam Tyson
PowerScore Staff
  • PowerScore Staff
  • Posts: 5153
  • Joined: Apr 14, 2011
|
#25584
Deep thinking, Alex! Glad it was a good mental workout, but I want to caution you that this level of analysis would be inappropriate when taking the actual test, due to the time constraints. Instead, use this deep thinking in untimed practice to enhance your understanding of the patterns used by the test authors and the concepts and strategies you need to deal with in order to quickly and accurately select the correct answer.

I would have to disagree with some of your premises here - there is nothing about this question that challenges any premises or asks that you to change anything. It's strictly a "what if" scenario, and a local question at that (so no "butterfly effect" problem).

Looking to your options in your question, #2 is the right answer in both cases - the right answer should "show what would have had to happen in Webster’s life in order for the modern critics’ current (as-is) view of him and his works to be correct" and in this type of question more generally "we try to imagine a hypothetical in which the observer’s current (erroneous) opinion could remain the same but would no longer be erroneous." Those are perfect descriptions!

Think of this as a causal argument - the author believes that Webster's not being influenced by the Elizabethan model of morality play causes the critics to be wrong about him. We want to find an answer choice that would remove that effect - we want the critics to be right. To do that, remove the cause - if Webster is influenced by the morality play (no cause) the critics are not wrong (no effect). Strengthening that causal argument by showing that when the cause is removed, the effect is removed, does not change anything or challenge anything - it just does what every strengthen argument is supposed to do, which is make the conclusion better.

Look at it again in this causal light and see if it doesn't make more sense now. Keep up the deep thinking while doing untimed practice in order to improve your overall understanding of the patterns and concepts, but remember that on test day it''s more about picking the best answer out of the (hopefully few) contenders, and not delving any deeper than necessary to get there.

Good luck!
 LaCrosse
  • Posts: 13
  • Joined: Apr 09, 2016
|
#25820
Hi Adam,

Thank you for aiding me in unraveling this puzzle. I have to admit I am not quite there yet.

Could you guide me into how you were able to infer from the wording of the question stem that the right answer needs to "show what would have had to happen in Webster’s life in order for the modern critics’ current (as-is) view of him and his works to be correct" as opposed to my other two options?
While thinking this through, I imagined myself as a painter with my client (let’s call him Mr. LSAT) pointing out two houses to me: one of them red, another green. Mr. LSAT says: “Make them both the same color!” How am I to know if Mr. LSAT wants two red houses, two green houses or two houses painted in some third color if that is all he tells me?

Let’s look at the critics’ argument from the causality perspective, as you’ve suggested. The critics (wrongly) believe that Webster has been influenced by the Elizabethan morality plays; such an influence would have caused him to compose plays in which purely good characters battle it out against the purely evil characters. The critics notice that Webster’s characters possess both good and bad traits and behave “inconsistently”, thus causing the critics to believe that Webster is an incompetent playwright who tried to write purely good and purely bad characters but failed miserably. In reality, Webster was influenced by the “more morally complicated Italian drama” which caused him to create more morally ambiguous characters on purpose.

As you’ve told me, Question 11 is to be interpreted as asking what would have had to happen in Webster’s life in order for the modern critics’ current (as-is) view of him and his works to be correct. The critics think that Elizabethan morality plays caused Webster to write in the same genre while doing a genuinely horrible job at it. Therefore, for the modern critics to be right in their assessment, Webster needs to be 1) be influenced by the Elizabethan morality play instead of the Italian drama; 2) actually compose morality plays; 3) genuinely try to come up with exclusively good and evil characters, yet be such an incompetent playwright that the characters still turn out to be morally ambiguous despite Webster’s best efforts otherwise.

Of course, this is a Strengthen question, not a Justify the Conclusion question: the correct answer choice does not need to provide us with airtight proof that the critics are right, it only needs to strengthen their argument, even if by as little as 1%. I can see how being “heavily influenced by the morality play” (answer choice C) might get Webster into composing morality plays on his own. But how does that cause Webster to start churning out the bad morality plays, the kind that are full of real, yet unintended inconsistencies and contradictions that the critics will interpret to mean that Webster is an incompetent playwright? If Webster commits to the Elizabethan model and sticks to its good vs. evil guidelines, the critics will be left with nothing to complain about. In a Strengthen question, can I get away with just strengthening one part of the argument and not the rest of it? For instance, if my conclusion is ”Alex will become an Olympic weightlifter and a leading authority on the writings of Nietzsche”, will the answer choice “Alex goes to the gym every day” strengthen that conclusion?

Thank you,

Alex
 Eric Ockert
PowerScore Staff
  • PowerScore Staff
  • Posts: 164
  • Joined: Sep 28, 2011
|
#25951
Hi Alex

Just to add to Adam's comments....

I would second Adam's caution about overthinking this question. What this really comes down to is knowing what the correct answer must do in order to be correct without getting too carried away on what it might do, but does not necessarily have to do.

The only thing this answer choice must do is to make their criticism "more valid." You are absolutely correct that this is not that strong of a standard, and an answer that makes their interpretation as little as 1% more likely could do the trick. So your prephrase here would be something to the effect of: "Which answer choice, if true, would make it more likely that the critics' interpretation is correct?"

The trick, then, is to nail down exactly what that interpretation is. We can see that interpretation in L44-46 where the author claims that the critics are "attributing the inconsistencies in a character's behavior to artistic incompetence on Webster's part." In other words, incompetence is the cause of the inconsistencies. So if we are to adjust our prephrase based on that specific language, we are now looking for: "Which answer choice, if true, makes it more likely that artistic incompetence caused Webster's inconsistencies?"

The author certainly disagrees with this explanation, and argues against it in the passage. One of the author's challenges to the critics' interpretation was to essentially suggest that Webster may have been primarily influenced by different source material than his contemporaries. In other words, the author is suggesting this difference in influence as a potential alternate cause of Webster's inconsistent characters as opposed to the critic's cause of artistic incompetence.

As you probably remember, one of the most common ways answer choices can strengthen causal arguments is to eliminate alternate causes. Answer choice (C) would eliminate the difference in influence as a possible explanation for Webster's inconsistencies. While this in no way proves the critics were correct, it does make their explanation more likely by eliminating an alternative.

And remember, the correct answer to this question did not have to eliminate an alternative. There are many other ways to strengthen this argument as well. But the correct answer did have to make the critics' explanation more likely and answer choice (C) certainly did that.

Now to the examples you cite:
1. If Mr. LSAT tells you solely to "make them both the same color" then all you have to do is make them both the same color. The key is that you do not know what color to paint the houses, and it doesn't matter. As long as they are the same color, you followed the instructions.

2. If your conclusion is "Alex will become an Olympic weightlifter and a leading authority on the writings of Nietzsche" then your answer choice that "Alex goes to the gym every day" would definitely strengthen that argument even though it never addresses Nietzsche. Remember, with Strengthen questions your prephrase is simply, "Which answer choice, if true, makes the conclusion in the argument more likely to be true?" While many correct answers to strengthen questions are very thorough, often going so far as to prove the conclusion, they in no way have to do this. As long as the answer improves the likelihood of the conclusion, no matter the degree, then that answer strengthens the argument.

Hope that helps!

Hope that helps!
 LaCrosse
  • Posts: 13
  • Joined: Apr 09, 2016
|
#26176
Hello Eric,

Thank you, this does help! There was definitely an “a-ha!” moment for me here as I suddenly saw this problem from a different angle and lots of things have started to make much more sense.
Big thanks to you and Adam for your wonderful explanations!

Best regards,

Alex

Get the most out of your LSAT Prep Plus subscription.

Analyze and track your performance with our Testing and Analytics Package.